Absolute Convergence of Homework Series: Real Parameter p

Click For Summary
The series \(\sum_{n=2}^{\infty} \ln \left(1+\frac{(-1)^n}{n^p}\right)\) converges absolutely for \(p > 1\) as shown by applying the limit comparison test. For \(0 < p \leq 1\), the series converges non-absolutely, which is supported by the behavior of the logarithmic function and the alternating series test. The limit comparison test indicates that the logarithmic terms approach zero when \(p\) is positive, suggesting convergence. The discussion highlights the need to analyze the series by separating even and odd terms to fully understand convergence behavior. Overall, the series exhibits distinct convergence properties based on the value of the real parameter \(p\).
dobry_den
Messages
113
Reaction score
0

Homework Statement


\sum_{n=2}^{\infty} \ln \left(1+\frac{(-1)^n}{n^p}\right)

p is a real parameter, determine when the series converges absolutely/non-absolutely

The Attempt at a Solution



I tried to do the limit \lim_{n\rightarrow \infty} \frac{\ln \left(1+\frac{(-1)^n}{n^p}\right)}{\frac{(-1)^n}{n^p}}, which is equal to one and this suggests that the series coverges if p is positive (limit comparison test). But then I'm not sure how to determine the absolute/non-absolute convergence. Could you help me please? Thanks very much in advance!
 
Last edited:
Physics news on Phys.org
Try to split the series into two parts - the even numbers and the odd ones and see if they both converge.
 
i made a mistake in the first post, the limit comparison test is applicable only to non-negative series. then the limit should be like:
\lim_{n\rightarrow \infty} \frac{\left|\ln \left(1+\frac{(-1)^n}{n^p}\right)\right|}{\frac{1}{n^p}}

which is equal to zero when p is positive. However, the lower series converges when p>1. Therefore, the original logarithm series converges absolutely for p>1.

The textbook then says that the series converges non-absolutely also for 1/2 < p <= 1. But I can't prove it - do you have any ideas?
 
Last edited:
Question: A clock's minute hand has length 4 and its hour hand has length 3. What is the distance between the tips at the moment when it is increasing most rapidly?(Putnam Exam Question) Answer: Making assumption that both the hands moves at constant angular velocities, the answer is ## \sqrt{7} .## But don't you think this assumption is somewhat doubtful and wrong?

Similar threads

  • · Replies 3 ·
Replies
3
Views
1K
  • · Replies 7 ·
Replies
7
Views
2K
  • · Replies 5 ·
Replies
5
Views
2K
  • · Replies 2 ·
Replies
2
Views
2K
Replies
5
Views
2K
  • · Replies 6 ·
Replies
6
Views
2K
  • · Replies 2 ·
Replies
2
Views
2K
  • · Replies 1 ·
Replies
1
Views
2K
  • · Replies 3 ·
Replies
3
Views
2K
  • · Replies 4 ·
Replies
4
Views
2K